67 of 100: Tram O'Rama

Logic Level 1

M A R T + T R A M R A M A \begin{array} {ccccc} \large & & \color{#D61F06}M & \color{#20A900}A & \color{#3D99F6}R & \color{#69047E}T\\ \large + & &\color{#69047E}T & \color{#3D99F6}R& \color{#20A900}A & \color{#D61F06}M \\ \hline \large & & \color{#3D99F6}R & \color{#20A900}A & \color{#D61F06}M & \color{#20A900}A \end{array}

If each letter represents a different nonzero digit, what is M ? \color{#D61F06}M\color{#333333}?

Another tough, algebraic cryptogram puzzle!

5 6 7 8 9

This section requires Javascript.
You are seeing this because something didn't load right. We suggest you, (a) try refreshing the page, (b) enabling javascript if it is disabled on your browser and, finally, (c) loading the non-javascript version of this page . We're sorry about the hassle.

16 solutions

A+R = A must be true when R=0 or there is a 1 at hand. R is non-zero.
So, A+R+1= A+10. or, R=9.

Now, T+M+1 = 9. Or, T+M= 8. So, A=8. Now it shapes as:

At tens digit, 9+8= 17, and we have noting at hand. So, M=7. Putting M=7 and T=1, we get a correct answer:

So, M=7

Nice solution!

William Huang - 3 years, 10 months ago

Yes - nice and simply done -using logic and basic number facts.

Katherine barker - 3 years, 10 months ago

Excellent. Many thanks.

julian grey - 3 years, 10 months ago

We note that the thousands column is M + T + { 0 1 = R M+T+{\color{#D61F06}\{_0^1}=R , where { 0 1 \color{#D61F06}\{_0^1 denotes the carry-forward of 1 or 0. This means that M + T + { 0 1 9 M+T + \{_0^1 \le 9 . We also note that the units column is also T + M = A T+M=A . This means that T + M 8 T+M \le 8 and M + T + 1 = R M+T+{\color{#D61F06}1} = R , implying that R = A + 1 R=A+1 . Also implying that in the hundreds and tens columns A + R = R + A > 10 A+R=R+A>10 . A + R + 1 = 10 + A \implies A+R+1=10+A R = 9 \implies R=9 , A = R 1 = 8 \implies A=R-1=8 . The tens column R + A = 10 + M R+A=10+M 9 + 8 = 10 + M \implies 9+8 = 10+M M = 7 \implies M = \boxed{7} . And the units column T + M = A T+M=A T + 7 = 8 \implies T+7=8 T = 1 \implies T=1 .

7 8 9 1 + 1 9 8 7 9 8 7 8 \begin{array} {ccccc} & \color{#D61F06}\boxed{7} & \color{#20A900}8 & \color{#3D99F6}9 & \color{#69047E}1 \\ + & \color{#69047E}1 & \color{#3D99F6}9 & \color{#20A900}8 & \color{#D61F06}\boxed{7} \\ \hline & \color{#3D99F6}9 & \color{#20A900}8 & \color{#D61F06}\boxed{7} & \color{#20A900}8 \end{array}

This solution is awesome!

William Huang - 3 years, 10 months ago
Aareyan Manzoor
Aug 5, 2017

notice that a maximum of 1 can carry over from the third digit to the second digit. so either A + R A m o d 10 R 0 m o d 10 R = 0 or A + R + 1 A m o d 10 R 1 9 m o d 10 R = 9 A+R\equiv A \mod{10}\to R\equiv 0 \mod{10}\to R=0 \\ \text{or}\\A+R+1\equiv A\mod{10}\to R\equiv -1\equiv 9 \mod{10} \to R=9 since R is nonzero, the first option is impossible meaning R = 9 R=9 . next we see a one carries over the first digit, meaining M + T + 1 = R M + T + 1 = 9 M + T = 8 M+T+1=R\to M+T+1= 9\to M+T=8 from the fourth digit T + M A m o d 10 8 = A T+M\equiv A\mod{10}\to 8=A . now the third digit says R + A M m o d 10 M 17 m o d 10 M = 7 R+A\equiv M \mod{10} \to M\equiv 17 \mod{10}\to M=\boxed{7} and ofcourse T = 8 M = 1 T=8-M=1 . the crypto gram is 7 8 9 1 + 1 9 8 7 9 8 7 8 \begin{array} {ccccc} \large & & \color{#D61F06}7 & \color{#20A900}8 & \color{#3D99F6}9 & \color{#69047E}1\\ \large + & &\color{#69047E}1 & \color{#3D99F6}9& \color{#20A900}8 & \color{#D61F06}7 \\ \hline \large & & \color{#3D99F6}9 & \color{#20A900}8& \color{#D61F06}7 & \color{#20A900}8 \end{array}

Well, I can already tell that this is going to be the featured solution.

William Huang - 3 years, 10 months ago

Your logic is perfect, but I would use "first", "second', "third" and "fourth" going from left to right when describing the digits of a number. The ones digit is the last digit, not the first digit.

Richard Desper - 3 years, 10 months ago

Log in to reply

thanks i have edited the solution

Aareyan Manzoor - 3 years, 10 months ago
Matt Doe
Aug 6, 2017

A straightforward solution in Javascript Outputs: 0 and 7

David Fairer
Aug 18, 2017

From the hundreds column with the digit A in the sum and also being the answer means that there must be a 'carry over' from the previous column so that 1 + R = 10. So R = 9. This 'carry over' + R to 'carry over' 1 on the thousands column gives that 1 + ( M + T) = R (=9). So M + T = 8 (= A from the 1sr column). And the first column did not have a carry over of 1. So from the 2nd column R + A is 9 + 8 = 17. So the M = 7 and there's 1 to carry over from column 2, which we knew already. And to get the last digit just for completeness from the 1st column again M + T = A, and M = 7, A = 8 gives T = 1. So the sum is 7,891 + 1,987 = 9,878 which is correct. And M has been seen to be 7. Regards, David

T + M = A T = M A R + A = 10 + M R + T + M = 10 + M R + T = 10 A + R + 1 = 10 + A R + 1 = 10 R = 9 T = 1 M + T + 1 = R M + T + 1 = 9 M + T = 8 M + T = A A = 8 M + 1 = 8 M = 7 T + M = A \\ T = M - A \\ \\ R + A = 10 + M \\ R + T + M = 10 + M \\ R + T = 10 \\ \\ A + R + 1 = 10 + A \\ R + 1 = 10 \\ R = 9 \\ \\ T = 1 \\ \\ M + T + 1 = R \\ M + T + 1 = 9 \\ M + T = 8 \\ \\ M + T = A \\ A = 8 \\ \\ M + 1 = 8 \\ \large M = \boxed{\color{#D61F06}7}

Mohammad Khaza
Aug 6, 2017

from, the left most column, we get= T + M = A T+M=A ....................[suppose]

again, from the right most column,we get= M + T = R M+T=R ............[suppose]

we know that, A + B = B + A A+B=B+A

so, we can say that M + T R M+T≠R as after doing some calculation ,it comes at the last.and, we can assume that, M + T = A M+T=A

and now,, A + R + 1 = 10 + A A+R+1=10+A

or, R = 9 R=9

again,M+T+1=9

or, M + T = A = 8 M+T=A=8

so, just doing the sum, R + A = 9 + 8 = 17 R+A=9+8=17

we have to take 7 for M and 10 for the next calculation.so, its clear that, M = 7 M=7

Tom Verhoeff
Aug 6, 2017

I used arithmetic modulo 11 to help me. Here is how. From the leftmost and rightmost column we infer R = A + 1 R = A + 1 (also see Arjen's solution). From the second column and R 0 R \ne 0 , we infer R = 9 R = 9 and there was a carry from the third column. Hence, A = 8 A = 8 . Taking everything m o d 11 {}\bmod 11 , we find M A + R T + T R + A M = R A + M A ( m o d 11 ) M - A + R - T + T - R + A - M = R - A + M - A \pmod{11} . This simplifies to 0 = R 2 A + M ( m o d 11 ) 0 = R -2A + M \pmod{11} . Using R , A = 9 , 8 R, A = 9, 8 , we find M = 7 ( m o d 11 ) M = 7 \pmod{11} , which implies M = 7 \fbox{M = 7} .

Compare the first and fourth columns. In both, T T and M M are added together, but they result in a different digit. This difference must be due to the fact that there is a carry-over from the second column to the first. Thus T + M = A , T + M + 1 = R , R = A + 1. T + M = A,\ \ T + M + 1 = R,\ \ R = A + 1. In the same way, the second and third column show that R + A = 10 + M , R + A + 1 = 10 + A , A = M + 1. R + A = 10 + M,\ \ R + A + 1 = 10 + A,\ \ A = M + 1. Since T + M = A T + M = A and 1 + M = A 1 + M = A , we know that T = 1 T = 1 .

From the second column we also see that A + R + 1 = A + 10 R = 9. A + R + 1 = A + 10\ \ \ \therefore\ \ \ \ R = 9. We immediately conclude that A = 8 A = 8 and M = 7 \boxed{M = 7} .

Syrous Marivani
Aug 6, 2017

A little investigation in the addition reveals that:

T + M = A, R + A = 10 + M 10 – T = R, 1 + A + R = 10 + A, R = 9, T = 1

1 + M + 1 = 9, M = 7, A = 8.

Therefore, M = 7.

Robert DeLisle
Aug 6, 2017

Now, is that really so "spiky"?

To get past two cacti shouldn't the solution require something that is more than one level deep?

Uros Stojkovic
Aug 6, 2017

Since addition of two four-digit numbers here yields also four-digit number, we can be sure that M + T + 1 < 10 M+T+1<10 , and hence M + T < 10 M+T<10 also. Then, we can with certainty write M + T = A M+T=A . From that we can see that M < A M<A and thus R + A = 10 + M R+A=10+M . Now, A + R + 1 = 10 + A R = 9 \begin{aligned} & A+R+1=10+A \\ & \rightarrow R=9\end{aligned} .

Next, M + T + 1 = 9 M + T = A = 8 \begin{aligned} & M+T+1=9 \\ & \rightarrow M+T=A=8 \end{aligned} .

From R + A = 9 + 8 = 10 + R R+A=9+8=10+R , we derive M = 7 M=7 .

1) Since A +R = M (hundreds) and R + A = A (tens), only one of these columns has a carry-over one from the prior column (tens or ones).

2) Since R is a digit (def: less than 10) and R+A=A there must be a carry over from the tens, such that R+1=10. Now we know R is 9.

3) Since R+A=M and we k ow there is no carry-over from the ones, M=A-1.

4) From the hundreths see T + M = R and from the ones : T+M=A. We know the ones do not have a carry over, so: T+M+1=A+1=R=9. That makes A=8.

Result: M=A-1=7

Avik Das
Aug 5, 2017

In the second column, A+R=A ,.
So, it is sure that, A+1=R.
Now, maximum value of (M+T)=8,as then A=8 and R=9.
In the third column,R+A=M,.
So, it is sure that, M+1=A.
So, T=1.
Now, from third column, (M+2)+(M+1)=M;i.e.(M+2)+(M+1)=M+10
or,M=7
So, the answer is 7







Sundar R
Aug 5, 2017

M + T= A

M + T + 1 = R

R = A+ 1

Looking at the third addition from the right, 2A + 2 = 10 + A (There is definitely a carryover in the R + A addition since M is smaller than A & R. So, A + R + 1 = 10 + A and R = A + 1)

A = 8

R= 9

M = 7

T= 1

Choose left = min {all answers}

Choose right = max {all answers}

Choose mid = (left+right)/2

So there mid=7

Answer mid

So you just picked the mean possible answer?

Richard Desper - 3 years, 10 months ago

What does this all mean?

Zach Abueg - 3 years, 10 months ago

Log in to reply

Usually I solve such puzzles with brute force using programming languages

I became really boring solving it that way

Solving it in a "proper" way is too hard for me

So I have tried to guess the answer

Because I can't produce truly random numbers I have thought about a simple procedure to produce pseudo random number

Nikita Mahilewets - 3 years, 10 months ago

0 pending reports

×

Problem Loading...

Note Loading...

Set Loading...